LSAT and Law School Admissions Forum

Get expert LSAT preparation and law school admissions advice from PowerScore Test Preparation.

 Administrator
PowerScore Staff
  • PowerScore Staff
  • Posts: 8929
  • Joined: Feb 02, 2011
|
#23166
Complete Question Explanation

Flaw in the Reasoning-SN. The correct answer choice is (A)

In this stimulus the conclusion does not follow from the premises because there is another conclusion that is equally, if not more, likely to be true. The argument does contain conditional reasoning, but it is not necessary to diagram this stimulus because it really is commonsense logic. The minister meets with the opposition and only his aide witnesses it. Somehow information from that meeting that hurts the minister gets leaked to a newspaper and he has to resign. The conclusion then states that the minister was brought down by his aide, not by any political opponent. However, it seems just as likely, if not more so, that the opposition leader, his opponent, leaked the story to force him to resign. Many of these Flaw questions use difficult wording to throw the test taker off, but the flaw is actually quite simple.

Answer choice (A): This is the correct answer choice, because there is an alternate conclusion (that the opposition leader leaked the information) that fits just as well with the evidence.

Answer choice (B) Don't get lost in the difficult wording in this answer choice. The argument says that the story could not be published without someone to leak the information, but there is no unwarranted assumption present. Do not choose this answer simply because it sounds difficult. In flaw questions, the most complicated answers are often tempting incorrect answers.

Answer choice (C) There are not two occasions to compare in the stimulus; therefore, this answer choice does not fit and is incorrect.

Answer choice (D) The conditional reasoning of the premises is relevant to the issue: someone had to leak the story to the newspaper and there were only three people there.

Answer choice (E) This answer does use the word "sufficient," so a test taker who sees the conditional reasoning present in the stimulus may be tempted. However, this answer choice seems to mainly be dealing with the resignation of the minister. The conclusion says the minister was "brought down" and the premises say that the newspaper story "forced the finance minister to resign." In real life, we might question whether the newspaper story was the entire reason, but, in the LSAT world, we can only use the information we are given. In this case, the premise clearly states that the newspaper story forced his resignation, so we must take that as a fact.
 reop6780
  • Posts: 265
  • Joined: Jul 27, 2013
|
#12362
The answer says the correct one is answer A.

I have no clue what "competing conclusion" exists.

What i focused on while reading was conditional reasoning.

"Without info..meeting..newspaper..not"
"No one witnessed...except the minister's aide"

In my head it looked like this,

newspaper written -> secret meeting

secret meeting -> minister's aide

I guess i was disappointed in that i could not find the answer that i expected
 Steve Stein
PowerScore Staff
  • PowerScore Staff
  • Posts: 1153
  • Joined: Apr 11, 2011
|
#12368
Hi,

This is an interesting question; I can see why you considered conditional diagramming, but I think a simple breakdown of the stimulus will make the flaw clear:

The information that got leaked had to come from someone who was at the meeting.
There were three people at the meeting:
1) the Minister, the Minister's Aide, and the Opposition Party leader.
2) the Minister's Aide
3) The OppositionParty Leader

The leak led to the resignation of the Minister. The author concludes that it must have been the Aide....any other conclusion you might draw?

Let me know your thoughts--thanks!

~Steve
 AnnBar
  • Posts: 33
  • Joined: Mar 24, 2017
|
#35239
Hello,

I am stuck on answer choice E. Even re-reading the question and explanations I am still between A and E. This was my thought process

treating evidence that a given action (newspaper story/leak) contributed to bringing about a certain effect (finance minister was brought down) as though that evidence established that the given action by itself was sufficient to bring about that effect.

what did I do wrong?

AB
 Emily Haney-Caron
PowerScore Staff
  • PowerScore Staff
  • Posts: 577
  • Joined: Jan 12, 2012
|
#35378
Hi AnnBar,

I think what happened here is that you didn't focus on what was a premise and what was the conclusion. Remember that premises are presented to us as true; we don't get to challenge them, even if we find them implausible, because the LSAT gives them to us as facts. You're trying to identify a flaw in a premise, but the premise isn't flawed; the premise is factual information, in this world. So, we take it for granted the newspaper story was the cause of the minister being brought down. Instead, you're looking for either a flaw in the conclusion, or a flaw in the relationship between the premises and conclusion. Does that make sense?
 AnnBar
  • Posts: 33
  • Joined: Mar 24, 2017
|
#35695
:0 it makes complete sense! Thank you
 PB410
  • Posts: 39
  • Joined: Apr 01, 2017
|
#36165
Hi, I was going over this question and managed to identify that there very well could have been an alternative explanation than to conclude that the minister's aide brought down the finance minister; however, I struggled to understand the meaning of the answer choices, which is a common problem I face. In not understanding the meaning of certain answer choices I miss correct answers, because the language of the test does not sync up with my own words. Even when reviewing types of flaws in the bible I still struggle. Is there any tips on how to better understand the wording in lsat answer choices?
 Francis O'Rourke
PowerScore Staff
  • PowerScore Staff
  • Posts: 471
  • Joined: Mar 10, 2017
|
#36207
Hi PB,

The best way I know for how to better understand the language that the Flaw in the Reasoning answer choices use is simply to practice more Flaw in the Reasoning questions. The language is difficult at first, but like learning a new language, you can gain fluency in this through practice and analysis. Fortunately, the word choice repeats fairly regularly, so there is a limited vocabulary that you need to gain fluency in to understand the answer choices.

When you come across a Flaw question like this one, you can add some terms to your vocabulary for the exam. In this case, you've now learned what the LSAT means when it describes a 'competing conclusion.' Every wrong answer you review will bring you one step closer to your goals, if you look at them as a chance to gain new understanding of the test.

This is a rather long process though, which makes Flaw in the Reasoning questions difficult for many people. Stick with it, and you will improve on them. For a slightly different approach to these questions you can read the following article by Nikki Siclunov:
How to Attack Flaw in the Reasoning Questions on the LSAT
 lathlee
  • Posts: 652
  • Joined: Apr 01, 2016
|
#42546
Hi. I couldn't recognize the flaw type. I know sufficient and necessary conditions are used to obtain the conclusion; I know it was a mistaken conclusion. but I don't think this is a mistaken-reversal or mistaken-negation error. may I get the type of flaw plz? Is it fair to say Use of Evidence error - Some evidence used to support the claim is true
 James Finch
PowerScore Staff
  • PowerScore Staff
  • Posts: 943
  • Joined: Sep 06, 2017
|
#42598
Hi Lathlee,

The conditional reasoning in this stimulus is actually a bit of a red herring, in that there is no flaw within the single conditional statement given. Instead the problem is that the stimulus erroneously applies another premise to the conditional reasoning, which leads to an unjustifiable conclusion. Let's break it down:

Premise: Newspaper story forces F.M. to resign :arrow: Someone at meeting between F.M and leader of opposition leaked story

Premise: F.M.'s trusted aide is only witness at meeting

Conclusion: Trusted aide leaked story

So we know that the story came out of the meeting, and that three people were present: the finance minister, his trusted aide, and the leader of the opposition. One (or more) of those three people must have leaked the story. Given that the finance minister was forced to resign as a result of the leak, we can safely exclude him as the leaker. This leaves both the aide and the leader of the opposition as possible sources. The stimulus jumps to the conclusion that the aide is the leaker, without bothering to rule out the leader of the opposition.

So there's our flaw: we have two potential candidates, but no other evidence pointing to one as more likely than the other. Answer choice (A) restates this flaw, by pointing out that we're left with a 50/50 possibility of either one of them being the leaker, but the stimulus's conclusion simply decides that it was the aide, not the leader of the opposition, without reason.

Hope this clears things up!

Get the most out of your LSAT Prep Plus subscription.

Analyze and track your performance with our Testing and Analytics Package.